Changing order of integration:$int_0^inftyint_{-infty}^{-y}f(x)mathrm dxmathrm...












2












$begingroup$



Why does $$int_{0}^{infty} int_{-infty}^{-y} f(x)mathrm dx mathrm dy Rightarrow int_{-infty}^{0} int_{0}^{-x} f(x) mathrm dy mathrm dx$$




The title is pretty self explanatory. I couldn't see how to properly change the order of the left integeral to the right one.



I'd love to hear your thoughts, thanks.










share|cite|improve this question











$endgroup$








  • 1




    $begingroup$
    Have you tried drawing the region?
    $endgroup$
    – Tom Himler
    Jan 14 at 21:36






  • 1




    $begingroup$
    The first thing that comes to mind is Paul's Online Math Notes. They're very good at explaining double integrals over general regions. tutorial.math.lamar.edu/Classes/CalcIII/DIGeneralRegion.aspx
    $endgroup$
    – Tom Himler
    Jan 14 at 21:46






  • 1




    $begingroup$
    Apply Fubini's theorem to $$int_{-infty}^{infty} int_{-infty}^{infty}f(x) , mathbf{1}_{{x leqslant -y }}mathbf{1}_{{x leqslant 0 }}, mathbf{1}_{{y geqslant 0 }},dx , dy$$
    $endgroup$
    – RRL
    Jan 14 at 21:57
















2












$begingroup$



Why does $$int_{0}^{infty} int_{-infty}^{-y} f(x)mathrm dx mathrm dy Rightarrow int_{-infty}^{0} int_{0}^{-x} f(x) mathrm dy mathrm dx$$




The title is pretty self explanatory. I couldn't see how to properly change the order of the left integeral to the right one.



I'd love to hear your thoughts, thanks.










share|cite|improve this question











$endgroup$








  • 1




    $begingroup$
    Have you tried drawing the region?
    $endgroup$
    – Tom Himler
    Jan 14 at 21:36






  • 1




    $begingroup$
    The first thing that comes to mind is Paul's Online Math Notes. They're very good at explaining double integrals over general regions. tutorial.math.lamar.edu/Classes/CalcIII/DIGeneralRegion.aspx
    $endgroup$
    – Tom Himler
    Jan 14 at 21:46






  • 1




    $begingroup$
    Apply Fubini's theorem to $$int_{-infty}^{infty} int_{-infty}^{infty}f(x) , mathbf{1}_{{x leqslant -y }}mathbf{1}_{{x leqslant 0 }}, mathbf{1}_{{y geqslant 0 }},dx , dy$$
    $endgroup$
    – RRL
    Jan 14 at 21:57














2












2








2


0



$begingroup$



Why does $$int_{0}^{infty} int_{-infty}^{-y} f(x)mathrm dx mathrm dy Rightarrow int_{-infty}^{0} int_{0}^{-x} f(x) mathrm dy mathrm dx$$




The title is pretty self explanatory. I couldn't see how to properly change the order of the left integeral to the right one.



I'd love to hear your thoughts, thanks.










share|cite|improve this question











$endgroup$





Why does $$int_{0}^{infty} int_{-infty}^{-y} f(x)mathrm dx mathrm dy Rightarrow int_{-infty}^{0} int_{0}^{-x} f(x) mathrm dy mathrm dx$$




The title is pretty self explanatory. I couldn't see how to properly change the order of the left integeral to the right one.



I'd love to hear your thoughts, thanks.







integration multivariable-calculus change-of-variable






share|cite|improve this question















share|cite|improve this question













share|cite|improve this question




share|cite|improve this question








edited Jan 14 at 21:44









Zacky

7,89511062




7,89511062










asked Jan 14 at 21:33









superuser123superuser123

48628




48628








  • 1




    $begingroup$
    Have you tried drawing the region?
    $endgroup$
    – Tom Himler
    Jan 14 at 21:36






  • 1




    $begingroup$
    The first thing that comes to mind is Paul's Online Math Notes. They're very good at explaining double integrals over general regions. tutorial.math.lamar.edu/Classes/CalcIII/DIGeneralRegion.aspx
    $endgroup$
    – Tom Himler
    Jan 14 at 21:46






  • 1




    $begingroup$
    Apply Fubini's theorem to $$int_{-infty}^{infty} int_{-infty}^{infty}f(x) , mathbf{1}_{{x leqslant -y }}mathbf{1}_{{x leqslant 0 }}, mathbf{1}_{{y geqslant 0 }},dx , dy$$
    $endgroup$
    – RRL
    Jan 14 at 21:57














  • 1




    $begingroup$
    Have you tried drawing the region?
    $endgroup$
    – Tom Himler
    Jan 14 at 21:36






  • 1




    $begingroup$
    The first thing that comes to mind is Paul's Online Math Notes. They're very good at explaining double integrals over general regions. tutorial.math.lamar.edu/Classes/CalcIII/DIGeneralRegion.aspx
    $endgroup$
    – Tom Himler
    Jan 14 at 21:46






  • 1




    $begingroup$
    Apply Fubini's theorem to $$int_{-infty}^{infty} int_{-infty}^{infty}f(x) , mathbf{1}_{{x leqslant -y }}mathbf{1}_{{x leqslant 0 }}, mathbf{1}_{{y geqslant 0 }},dx , dy$$
    $endgroup$
    – RRL
    Jan 14 at 21:57








1




1




$begingroup$
Have you tried drawing the region?
$endgroup$
– Tom Himler
Jan 14 at 21:36




$begingroup$
Have you tried drawing the region?
$endgroup$
– Tom Himler
Jan 14 at 21:36




1




1




$begingroup$
The first thing that comes to mind is Paul's Online Math Notes. They're very good at explaining double integrals over general regions. tutorial.math.lamar.edu/Classes/CalcIII/DIGeneralRegion.aspx
$endgroup$
– Tom Himler
Jan 14 at 21:46




$begingroup$
The first thing that comes to mind is Paul's Online Math Notes. They're very good at explaining double integrals over general regions. tutorial.math.lamar.edu/Classes/CalcIII/DIGeneralRegion.aspx
$endgroup$
– Tom Himler
Jan 14 at 21:46




1




1




$begingroup$
Apply Fubini's theorem to $$int_{-infty}^{infty} int_{-infty}^{infty}f(x) , mathbf{1}_{{x leqslant -y }}mathbf{1}_{{x leqslant 0 }}, mathbf{1}_{{y geqslant 0 }},dx , dy$$
$endgroup$
– RRL
Jan 14 at 21:57




$begingroup$
Apply Fubini's theorem to $$int_{-infty}^{infty} int_{-infty}^{infty}f(x) , mathbf{1}_{{x leqslant -y }}mathbf{1}_{{x leqslant 0 }}, mathbf{1}_{{y geqslant 0 }},dx , dy$$
$endgroup$
– RRL
Jan 14 at 21:57










2 Answers
2






active

oldest

votes


















3












$begingroup$

The easiest way to perform a change of the order of integration in the multivariable setting is via Iverson's bracket. This is the indicator function such that $$[P] =begin{cases} 1 & P text{ is true}\
0 & text{else}.end{cases}$$
The change of variable arises from reinterpreting the system of inequalities (see below).



With the Iverson notation, one can remove the boundaries from the integral and implement it in the integrand, i.e.,
$$int_{0}^{infty} int_{-infty}^{-y} f(x,y)dx ,dy =iint_{mathbb{R}^2} f(x,y)Bigl[(ygeq 0) text{ and } (xleq-y) Bigr]dx, dy ,.$$



Now in order to perform the change of the order of integration, you have to reinterpret Iverson's bracket. You have to figure out what condition $$P=(ygeq0) text{ and } (xleq-y)$$ poses on $x$ first.



The maximal value that $x$ can achieve is $0$ (when $y=0$). The second condition in $P$ is equivalent to
$$ x leq -y Leftrightarrow y leq -x ,.$$
The first condition demands that $y>0$. Together, we have that
lf that $P$ is equivalent to
$$ P Leftrightarrow (x<0) text{ and } (0 < y < -x ),.$$



So we find
$$int_{0}^{infty} int_{-infty}^{-y} f(x,y)dx, dy =iint_{mathbb{R}^2} f(x,y)Bigl[(x<0) text{ and } (0 < y < -x )Bigr]dx ,dy = int_{-infty}^0 int_{0}^{-x} f(x,y) ,dy,dx ,.$$






share|cite|improve this answer









$endgroup$





















    2












    $begingroup$

    The inequalities given with the first double integral are $0leq y leq infty$ and $-infty leq x leq -y$.



    If you plot the funtions and the regions you get the region:



    enter image description here



    As you can see, if you we let the $x$ be constant limits instead of $y$ the same region can be defined as $-infty leq x leq 0$ and $0leq y leq -x$. Which gives you the bounds in your question.






    share|cite|improve this answer









    $endgroup$













      Your Answer





      StackExchange.ifUsing("editor", function () {
      return StackExchange.using("mathjaxEditing", function () {
      StackExchange.MarkdownEditor.creationCallbacks.add(function (editor, postfix) {
      StackExchange.mathjaxEditing.prepareWmdForMathJax(editor, postfix, [["$", "$"], ["\\(","\\)"]]);
      });
      });
      }, "mathjax-editing");

      StackExchange.ready(function() {
      var channelOptions = {
      tags: "".split(" "),
      id: "69"
      };
      initTagRenderer("".split(" "), "".split(" "), channelOptions);

      StackExchange.using("externalEditor", function() {
      // Have to fire editor after snippets, if snippets enabled
      if (StackExchange.settings.snippets.snippetsEnabled) {
      StackExchange.using("snippets", function() {
      createEditor();
      });
      }
      else {
      createEditor();
      }
      });

      function createEditor() {
      StackExchange.prepareEditor({
      heartbeatType: 'answer',
      autoActivateHeartbeat: false,
      convertImagesToLinks: true,
      noModals: true,
      showLowRepImageUploadWarning: true,
      reputationToPostImages: 10,
      bindNavPrevention: true,
      postfix: "",
      imageUploader: {
      brandingHtml: "Powered by u003ca class="icon-imgur-white" href="https://imgur.com/"u003eu003c/au003e",
      contentPolicyHtml: "User contributions licensed under u003ca href="https://creativecommons.org/licenses/by-sa/3.0/"u003ecc by-sa 3.0 with attribution requiredu003c/au003e u003ca href="https://stackoverflow.com/legal/content-policy"u003e(content policy)u003c/au003e",
      allowUrls: true
      },
      noCode: true, onDemand: true,
      discardSelector: ".discard-answer"
      ,immediatelyShowMarkdownHelp:true
      });


      }
      });














      draft saved

      draft discarded


















      StackExchange.ready(
      function () {
      StackExchange.openid.initPostLogin('.new-post-login', 'https%3a%2f%2fmath.stackexchange.com%2fquestions%2f3073785%2fchanging-order-of-integration-int-0-infty-int-infty-yfx-mathrm-dx-ma%23new-answer', 'question_page');
      }
      );

      Post as a guest















      Required, but never shown

























      2 Answers
      2






      active

      oldest

      votes








      2 Answers
      2






      active

      oldest

      votes









      active

      oldest

      votes






      active

      oldest

      votes









      3












      $begingroup$

      The easiest way to perform a change of the order of integration in the multivariable setting is via Iverson's bracket. This is the indicator function such that $$[P] =begin{cases} 1 & P text{ is true}\
      0 & text{else}.end{cases}$$
      The change of variable arises from reinterpreting the system of inequalities (see below).



      With the Iverson notation, one can remove the boundaries from the integral and implement it in the integrand, i.e.,
      $$int_{0}^{infty} int_{-infty}^{-y} f(x,y)dx ,dy =iint_{mathbb{R}^2} f(x,y)Bigl[(ygeq 0) text{ and } (xleq-y) Bigr]dx, dy ,.$$



      Now in order to perform the change of the order of integration, you have to reinterpret Iverson's bracket. You have to figure out what condition $$P=(ygeq0) text{ and } (xleq-y)$$ poses on $x$ first.



      The maximal value that $x$ can achieve is $0$ (when $y=0$). The second condition in $P$ is equivalent to
      $$ x leq -y Leftrightarrow y leq -x ,.$$
      The first condition demands that $y>0$. Together, we have that
      lf that $P$ is equivalent to
      $$ P Leftrightarrow (x<0) text{ and } (0 < y < -x ),.$$



      So we find
      $$int_{0}^{infty} int_{-infty}^{-y} f(x,y)dx, dy =iint_{mathbb{R}^2} f(x,y)Bigl[(x<0) text{ and } (0 < y < -x )Bigr]dx ,dy = int_{-infty}^0 int_{0}^{-x} f(x,y) ,dy,dx ,.$$






      share|cite|improve this answer









      $endgroup$


















        3












        $begingroup$

        The easiest way to perform a change of the order of integration in the multivariable setting is via Iverson's bracket. This is the indicator function such that $$[P] =begin{cases} 1 & P text{ is true}\
        0 & text{else}.end{cases}$$
        The change of variable arises from reinterpreting the system of inequalities (see below).



        With the Iverson notation, one can remove the boundaries from the integral and implement it in the integrand, i.e.,
        $$int_{0}^{infty} int_{-infty}^{-y} f(x,y)dx ,dy =iint_{mathbb{R}^2} f(x,y)Bigl[(ygeq 0) text{ and } (xleq-y) Bigr]dx, dy ,.$$



        Now in order to perform the change of the order of integration, you have to reinterpret Iverson's bracket. You have to figure out what condition $$P=(ygeq0) text{ and } (xleq-y)$$ poses on $x$ first.



        The maximal value that $x$ can achieve is $0$ (when $y=0$). The second condition in $P$ is equivalent to
        $$ x leq -y Leftrightarrow y leq -x ,.$$
        The first condition demands that $y>0$. Together, we have that
        lf that $P$ is equivalent to
        $$ P Leftrightarrow (x<0) text{ and } (0 < y < -x ),.$$



        So we find
        $$int_{0}^{infty} int_{-infty}^{-y} f(x,y)dx, dy =iint_{mathbb{R}^2} f(x,y)Bigl[(x<0) text{ and } (0 < y < -x )Bigr]dx ,dy = int_{-infty}^0 int_{0}^{-x} f(x,y) ,dy,dx ,.$$






        share|cite|improve this answer









        $endgroup$
















          3












          3








          3





          $begingroup$

          The easiest way to perform a change of the order of integration in the multivariable setting is via Iverson's bracket. This is the indicator function such that $$[P] =begin{cases} 1 & P text{ is true}\
          0 & text{else}.end{cases}$$
          The change of variable arises from reinterpreting the system of inequalities (see below).



          With the Iverson notation, one can remove the boundaries from the integral and implement it in the integrand, i.e.,
          $$int_{0}^{infty} int_{-infty}^{-y} f(x,y)dx ,dy =iint_{mathbb{R}^2} f(x,y)Bigl[(ygeq 0) text{ and } (xleq-y) Bigr]dx, dy ,.$$



          Now in order to perform the change of the order of integration, you have to reinterpret Iverson's bracket. You have to figure out what condition $$P=(ygeq0) text{ and } (xleq-y)$$ poses on $x$ first.



          The maximal value that $x$ can achieve is $0$ (when $y=0$). The second condition in $P$ is equivalent to
          $$ x leq -y Leftrightarrow y leq -x ,.$$
          The first condition demands that $y>0$. Together, we have that
          lf that $P$ is equivalent to
          $$ P Leftrightarrow (x<0) text{ and } (0 < y < -x ),.$$



          So we find
          $$int_{0}^{infty} int_{-infty}^{-y} f(x,y)dx, dy =iint_{mathbb{R}^2} f(x,y)Bigl[(x<0) text{ and } (0 < y < -x )Bigr]dx ,dy = int_{-infty}^0 int_{0}^{-x} f(x,y) ,dy,dx ,.$$






          share|cite|improve this answer









          $endgroup$



          The easiest way to perform a change of the order of integration in the multivariable setting is via Iverson's bracket. This is the indicator function such that $$[P] =begin{cases} 1 & P text{ is true}\
          0 & text{else}.end{cases}$$
          The change of variable arises from reinterpreting the system of inequalities (see below).



          With the Iverson notation, one can remove the boundaries from the integral and implement it in the integrand, i.e.,
          $$int_{0}^{infty} int_{-infty}^{-y} f(x,y)dx ,dy =iint_{mathbb{R}^2} f(x,y)Bigl[(ygeq 0) text{ and } (xleq-y) Bigr]dx, dy ,.$$



          Now in order to perform the change of the order of integration, you have to reinterpret Iverson's bracket. You have to figure out what condition $$P=(ygeq0) text{ and } (xleq-y)$$ poses on $x$ first.



          The maximal value that $x$ can achieve is $0$ (when $y=0$). The second condition in $P$ is equivalent to
          $$ x leq -y Leftrightarrow y leq -x ,.$$
          The first condition demands that $y>0$. Together, we have that
          lf that $P$ is equivalent to
          $$ P Leftrightarrow (x<0) text{ and } (0 < y < -x ),.$$



          So we find
          $$int_{0}^{infty} int_{-infty}^{-y} f(x,y)dx, dy =iint_{mathbb{R}^2} f(x,y)Bigl[(x<0) text{ and } (0 < y < -x )Bigr]dx ,dy = int_{-infty}^0 int_{0}^{-x} f(x,y) ,dy,dx ,.$$







          share|cite|improve this answer












          share|cite|improve this answer



          share|cite|improve this answer










          answered Jan 14 at 22:00









          FabianFabian

          20k3774




          20k3774























              2












              $begingroup$

              The inequalities given with the first double integral are $0leq y leq infty$ and $-infty leq x leq -y$.



              If you plot the funtions and the regions you get the region:



              enter image description here



              As you can see, if you we let the $x$ be constant limits instead of $y$ the same region can be defined as $-infty leq x leq 0$ and $0leq y leq -x$. Which gives you the bounds in your question.






              share|cite|improve this answer









              $endgroup$


















                2












                $begingroup$

                The inequalities given with the first double integral are $0leq y leq infty$ and $-infty leq x leq -y$.



                If you plot the funtions and the regions you get the region:



                enter image description here



                As you can see, if you we let the $x$ be constant limits instead of $y$ the same region can be defined as $-infty leq x leq 0$ and $0leq y leq -x$. Which gives you the bounds in your question.






                share|cite|improve this answer









                $endgroup$
















                  2












                  2








                  2





                  $begingroup$

                  The inequalities given with the first double integral are $0leq y leq infty$ and $-infty leq x leq -y$.



                  If you plot the funtions and the regions you get the region:



                  enter image description here



                  As you can see, if you we let the $x$ be constant limits instead of $y$ the same region can be defined as $-infty leq x leq 0$ and $0leq y leq -x$. Which gives you the bounds in your question.






                  share|cite|improve this answer









                  $endgroup$



                  The inequalities given with the first double integral are $0leq y leq infty$ and $-infty leq x leq -y$.



                  If you plot the funtions and the regions you get the region:



                  enter image description here



                  As you can see, if you we let the $x$ be constant limits instead of $y$ the same region can be defined as $-infty leq x leq 0$ and $0leq y leq -x$. Which gives you the bounds in your question.







                  share|cite|improve this answer












                  share|cite|improve this answer



                  share|cite|improve this answer










                  answered Jan 14 at 22:45









                  Tom HimlerTom Himler

                  945314




                  945314






























                      draft saved

                      draft discarded




















































                      Thanks for contributing an answer to Mathematics Stack Exchange!


                      • Please be sure to answer the question. Provide details and share your research!

                      But avoid



                      • Asking for help, clarification, or responding to other answers.

                      • Making statements based on opinion; back them up with references or personal experience.


                      Use MathJax to format equations. MathJax reference.


                      To learn more, see our tips on writing great answers.




                      draft saved


                      draft discarded














                      StackExchange.ready(
                      function () {
                      StackExchange.openid.initPostLogin('.new-post-login', 'https%3a%2f%2fmath.stackexchange.com%2fquestions%2f3073785%2fchanging-order-of-integration-int-0-infty-int-infty-yfx-mathrm-dx-ma%23new-answer', 'question_page');
                      }
                      );

                      Post as a guest















                      Required, but never shown





















































                      Required, but never shown














                      Required, but never shown












                      Required, but never shown







                      Required, but never shown

































                      Required, but never shown














                      Required, but never shown












                      Required, but never shown







                      Required, but never shown







                      Popular posts from this blog

                      Human spaceflight

                      Can not write log (Is /dev/pts mounted?) - openpty in Ubuntu-on-Windows?

                      File:DeusFollowingSea.jpg